USMLE NBME 18 - Questions and Answers - Discussions & Explanations

This forum made possible through the generous support of SDN members, donors, and sponsors. Thank you.

TheAberrantGene

Full Member
7+ Year Member
Joined
Nov 26, 2015
Messages
37
Reaction score
30
NBME 18 has been released and is available on regular and extended feedback.
I will be taking it fairly soon as my exam is around the corner.
Let's continue the great trend on this forum and start a discussion once people start taking it,

Best of luck fellas ! :)

Members don't see this ad.
 
  • Like
Reactions: 3 users
thanks, how do we see this leukemoid reaction ?

The fact she has a history of 6 hrs of fever and shaking chills + leukocytosis suggests you this is an acute inflammatory reaction.
You can see she has nitrites and leukocyte esterase(Gram - bacteria) in urine and flank pain→ acute pyelonephritis.

a Leukemoid reaction is just an acute inflammatory reaction= ↑WBC count and ↑neutrophils, also ↑leukocyte alkaline phosphatase
 
  • Like
Reactions: 2 users
Ugh, thank you...I was leaning towards that but with neuropathy I figured if it were motility then they'd be constipated. :(
The pathogenesis of diabetic diarrhea includes reduced gastrointestinal motility, reduced receptor mediated fluid absorption, bacterial overgrowth, pancreatic insufficiency, coexistent celiac disease, and abnormalities in bile salt metabolism
http://www.medscape.com/viewarticle/473205_5
 
  • Like
Reactions: 1 users
1. A 7 YO boy is about undergo an appendectomy. An intravenous catheter needs to be inserted, but the patient is fearful of being stuck with needle. The most appropriate anesthesia administered by mask to anesthetized this patient quickly would have which of the following characteristics ?
a. High blood solubility
b. High cerebrospinal fluid solubility
c. High lipid solubility (WRONG)
d. Low blood solubility
e. Low lipid solubility

2. A 27 YO primigravid woman at 18 weeks gestation comes to physician for routine prenatal examination. The uterus consistent in size with 18 week gestation. Ultrasonography shows a male fetus. The collecting system and pelvis of the left kidney is dilated and the renal cortex appears compressed. The left and right ureters are not dilated. The right kidney appears normal. Amniotic fluid volume is normal. Which causing renal finding in this fetus ?
a. Abnormal ureter insertion into bladder wall
b. Agenesis of upper urinary tract
c. Congenital urethral stricture (WRONG)
d. Ectopic ureter
e. Incomplete recanalization of proximal ureter
f. Posterior urethral valves

3. A 25 yo man just returned from work as worker from Africa begins oral chloroquine therapy for malaria caused by Plasmodium vivax. His initial therapeutic response is good, but he develops recurrent parasitemia 2 months later. Which of the following best explains the recurrence ?
a. Chloroquine is ineffective as oral therapy for P. vivax malaria (WRONG)
b. Chloroquine is ineffective on the exoerythrocytic malaria tissue stages
c. Chloroquine is only effective against P. vivax when combined with metronidazole
d. The patient has a second previously occult malaria infection
e. The patient is inefected with a chloroquine-resistant strain of P.vivax

4. A 57 YO alcoholism dies of Klebsiella Pneu. Several abscess cavities filled with purulent exudate are found at autopsy. Which of the following patterns of necrosis would be found in lung tissue within these cavities ?
a. Caseous (is ithis ?? )
b. Fat
c. Fibrinoid
d. Gangrenous
e. Liquefaction

5. A 6 year old boy is brought tp physician by his mother because of 5 days Hx if intermitten vomiting and 3 month Hx of progressive clumliness. His mother says that he can no longer ride his bicycle and has difficulty getting in and out of the car. Funduscopy examination shows bilateral papilledema. Neurologic examination shows impaired upward gaze and pupil response to the right. Sensation is intact. He walks with shuffling gait. A CT scan of the head shows enlarged lateral and third ventricle and 2 cm mass. Where location of the mass ?
a. Cerebellar hemisphere
b. Frontal lobe
c. Lateral medulla (is this ??)
d. Pineal gland
 
Members don't see this ad :)
1. A 7 YO boy is about undergo an appendectomy. An intravenous catheter needs to be inserted, but the patient is fearful of being stuck with needle. The most appropriate anesthesia administered by mask to anesthetized this patient quickly would have which of the following characteristics ?
a. High blood solubility
b. High cerebrospinal fluid solubility
c. High lipid solubility (WRONG)
d. Low blood solubility
e. Low lipid solubility

2. A 27 YO primigravid woman at 18 weeks gestation comes to physician for routine prenatal examination. The uterus consistent in size with 18 week gestation. Ultrasonography shows a male fetus. The collecting system and pelvis of the left kidney is dilated and the renal cortex appears compressed. The left and right ureters are not dilated. The right kidney appears normal. Amniotic fluid volume is normal. Which causing renal finding in this fetus ?
a. Abnormal ureter insertion into bladder wall
b. Agenesis of upper urinary tract
c. Congenital urethral stricture (WRONG)
d. Ectopic ureter
e. Incomplete recanalization of proximal ureter
f. Posterior urethral valves

3. A 25 yo man just returned from work as worker from Africa begins oral chloroquine therapy for malaria caused by Plasmodium vivax. His initial therapeutic response is good, but he develops recurrent parasitemia 2 months later. Which of the following best explains the recurrence ?
a. Chloroquine is ineffective as oral therapy for P. vivax malaria (WRONG)
b. Chloroquine is ineffective on the exoerythrocytic malaria tissue stages
c. Chloroquine is only effective against P. vivax when combined with metronidazole
d. The patient has a second previously occult malaria infection
e. The patient is inefected with a chloroquine-resistant strain of P.vivax

4. A 57 YO alcoholism dies of Klebsiella Pneu. Several abscess cavities filled with purulent exudate are found at autopsy. Which of the following patterns of necrosis would be found in lung tissue within these cavities ?
a. Caseous (is ithis ?? )
b. Fat
c. Fibrinoid
d. Gangrenous
e. Liquefaction

5. A 6 year old boy is brought tp physician by his mother because of 5 days Hx if intermitten vomiting and 3 month Hx of progressive clumliness. His mother says that he can no longer ride his bicycle and has difficulty getting in and out of the car. Funduscopy examination shows bilateral papilledema. Neurologic examination shows impaired upward gaze and pupil response to the right. Sensation is intact. He walks with shuffling gait. A CT scan of the head shows enlarged lateral and third ventricle and 2 cm mass. Where location of the mass ?
a. Cerebellar hemisphere
b. Frontal lobe
c. Lateral medulla (is this ??)
d. Pineal gland


1. D (the lower the blood:gas coefficient, the faster the induction rate)
2. I think I put E, only one that made sense to me since the right kidney is fine and the left ureter isn't dilated.
3. B
4. E
5. D

(Someone correct me if I'm wrong on the first 2 cause my memory is shot)
 
can somebody post this question and options ?

sec 1
10. 50 yo man hospital pulm embolus tx iv heparin 24h later warfarin added. on day 2 ptt 52 sec (control 26 sec) an pt 12sec (control 12.2 sec INR 1) Which best explaantion for normal PT and INR of pt?

a.heparin warfairn interaction

b.hereditary resistence to heparin

c.hereditary resistance to warfarin

d.long half life of factor 2 prothrombin

e.too low dose heparin

f.too low dose warfarin

g. undetected liver disease
 
  • Like
Reactions: 1 user
this one from section 3 , last question

3.50 during a clinical study of calcium and phosphorus metabolism, a 50 50 yo man undergoes series of lab studies. His serum Ca, PO4 and PTH are normal.He is given infusion 2 g Calcium chloride over 2 hours. His serum ca concentration now is 11.5mg/dl. Compared with pre infusion levels, the serum concentration of which of the following substances is likely to be increased at this time?

7 dehydrocholesterol

1,25 dihydroxycholecalciferol (wroong)

24,25 dihydroxycholecalciferol

previtamin d3

vitamin d3
 
sec 1
10. 50 yo man hospital pulm embolus tx iv heparin 24h later warfarin added. on day 2 ptt 52 sec (control 26 sec) an pt 12sec (control 12.2 sec INR 1) Which best explaantion for normal PT and INR of pt?

a.heparin warfairn interaction

b.hereditary resistence to heparin

c.hereditary resistance to warfarin

d.long half life of factor 2 prothrombin

e.too low dose heparin

f.too low dose warfarin

g. undetected liver disease


That's what it was...I couldn't remember what the option was, definitely D with my explanation earlier.
 
  • Like
Reactions: 1 user
this one from section 3 , last question

3.50 during a clinical study of calcium and phosphorus metabolism, a 50 50 yo man undergoes series of lab studies. His serum Ca, PO4 and PTH are normal.He is given infusion 2 g Calcium chloride over 2 hours. His serum ca concentration now is 11.5mg/dl. Compared with pre infusion levels, the serum concentration of which of the following substances is likely to be increased at this time?

7 dehydrocholesterol

1,25 dihydroxycholecalciferol (wroong)

24,25 dihydroxycholecalciferol

previtamin d3

vitamin d3

I got this wrong but I *think* it's 24, 25 dihydroxy vit D. My reasoning being hypercalcemia would cause negative feedback on PTH therefore decreasing PTH ----> decreased conversion to 1,25 dihydroxy vit D by 1 alpha hydroxylase in kidneys and enhanced conversion to the metabolite 24,25 dihydroxy vit D instead.
 
Dead space has to do with areas that are ventilated but not perfused (more or less). A shunt is the opposite-- not ventilated, but perfused. D should be the answer.

If everything else stays the same, except for the decreased number of measurements (smaller sample size), the width of the confidence interval will increase. Two ways to get this answer: look at the formula for a confidence interval or use logic.

The formula: x-bar +/- (t critical)*(standard error of the mean) where standard error of the mean is equal to the sample standard deviation, s, divided by the square root of the sample size, n. In the first case, n=20 so 8/(sq.rt.20) is smaller than 8/(sq.rt. 4). Therefore, when n is reduced from 20 to 4, we can see the mean has a larger value added/subtracted from it, making the interval wider when n=4 instead of 20.

The other way to approach that is from a qualitative perspective. The width of a confidence interval represents our uncertainty concerning the estimate. If we have a larger sample, we should be less uncertain about our estimate (narrower interval). Similarly, if we have a smaller sample, we will be more uncertain about our estimate (again, assuming that the only change is sample size). This should make sense, because we would like to have all possible information (no uncertainty), but we can't usually get all the information (so we're left with uncertainty). Simply put, larger samples (more information) get us closer to the truth and reduce our uncertainty about our estimate of that truth (the mean, in this case).
B should be correct.

Logic takes you far on step 1
 
  • Like
Reactions: 1 user
I got this wrong but I *think* it's 24, 25 dihydroxy vit D. My reasoning being hypercalcemia would cause negative feedback on PTH therefore decreasing PTH ----> decreased conversion to 1,25 dihydroxy vit D by 1 alpha hydroxylase in kidneys and enhanced conversion to the metabolite 24,25 dihydroxy vit D instead.


well your logic makes sense to me! thanks:)
 
1.A 63 year old man comes to physician because of abnormal blood pressure reading taking 1 week ago. No history of major illness. ^ months ago BP was 135/85 today it is 170/98. Bruit heard over left renal artery. Spiral ct shows left arterial stenosis. Which of the following sets of changes will be seen for TPR plasma renin activity and serum aldosterone concentraion.

2.Previously healthy 35 year old woman comes to the physician because of 3 month history of progressive shortness of breath with exertion. Respiration are 26/min. Physical exam shows Jugular venous distention with a prominent a wave. Lungs are clear to auscultation. Cardiac exam shows loud pulmonic component of s2 and right sided s4. This patients condition most likely involves an increase pulm expression of which of the following?
A) Endothelin-1
B) Nitric oxide
c) prostacylin synthase
d) thrombomondulin
e) voltage gated K channels

3.A 50 year old woman with COPD comes to physician of 3 month history of progressive shortness of breath. Physical exam shows jugular venous distention. A loud pulmonary component of s2 is heard. Pulmonary function tests shows FEV1:FVC ratio of 20% and a severely decreased diffusing capacity of carbon monoxide. based on these findings which of the following is most likely decreased in patients pulmonary vascular smooth muscle.
a) adventitial collagen matrix deposition
b. cytosolic phosphlipase A2 activity
c endothelial nitric oxide synthase production
d endothelin expression
e) voltage gated calcium channel alpha subunit production

4.6 year old girl is brought to ED by dad cause of 15 min history of shortness of breath. She was diagnosed with throat tumor 3 years ago. Respiration are 32/min. Exam shows nasal flaring. Larygocopy shows multiple raised finger shaped lesions arising from the vocal cords and epiglottis and partially occluding the larynx. The lesions are excised. Micro exam shows finger shaped fibrovascular cores lined with benign squamous epithelium. Which of the following is the most likely virus.
a) CMV
b) EBV
C) HSV1
d)HSV2)
e) HPV type 6
 
section 1 question 30 (I summerized it):

elderly admitted with pyelonephritis, given ciprofloxacin, on day 3 her symptoms resolved but on day 5 she started to have watery diarrhea and right lower quadrant tenderness. her temp. is 38. occult stool blood is negative. rest is not remarkable.
next step in diagnosis?

a- send for ova/parasites
b- 24-hour stool fat
c- send for c.difficile toxin
d- stool culture
e- barium enema
f- stop cipro for 3 days and reassess
 
Members don't see this ad :)
section 1 question 30 (I summerized it):

elderly admitted with pyelonephritis, given ciprofloxacin, on day 3 her symptoms resolved but on day 5 she started to have watery diarrhea and right lower quadrant tenderness. her temp. is 38. occult stool blood is negative. rest is not remarkable.
next step in diagnosis?

a- send for ova/parasites
b- 24-hour stool fat
c- send for c.difficile toxin
d- stool culture
e- barium enema
f- stop cipro for 3 days and reassess
CCC
 
2.Previously healthy 35 year old woman comes to the physician because of 3 month history of progressive shortness of breath with exertion. Respiration are 26/min. Physical exam shows Jugular venous distention with a prominent a wave. Lungs are clear to auscultation. Cardiac exam shows loud pulmonic component of s2 and right sided s4. This patients condition most likely involves an increase pulm expression of which of the following?
A) Endothelin-1
B) Nitric oxide
c) prostacylin synthase
d) thrombomondulin
e) voltage gated K channels


4.6 year old girl is brought to ED by dad cause of 15 min history of shortness of breath. She was diagnosed with throat tumor 3 years ago. Respiration are 32/min. Exam shows nasal flaring. Larygocopy shows multiple raised finger shaped lesions arising from the vocal cords and epiglottis and partially occluding the larynx. The lesions are excised. Micro exam shows finger shaped fibrovascular cores lined with benign squamous epithelium. Which of the following is the most likely virus.
a) CMV
b) EBV
C) HSV1
d)HSV2)
e) HPV type 6

2) Exam suggests pulmonary hypertension, which is associated with excess pulmonic expression of endothelin-1. Treat with bosentan, definitive treatment is lung transplant.

4) laryngeal papillomatosis due to HPV 6
 
1.A 63 year old man comes to physician because of abnormal blood pressure reading taking 1 week ago. No history of major illness. ^ months ago BP was 135/85 today it is 170/98. Bruit heard over left renal artery. Spiral ct shows left arterial stenosis. Which of the following sets of changes will be seen for TPR plasma renin activity and serum aldosterone concentraion.

2.Previously healthy 35 year old woman comes to the physician because of 3 month history of progressive shortness of breath with exertion. Respiration are 26/min. Physical exam shows Jugular venous distention with a prominent a wave. Lungs are clear to auscultation. Cardiac exam shows loud pulmonic component of s2 and right sided s4. This patients condition most likely involves an increase pulm expression of which of the following?
A) Endothelin-1 Correct
B) Nitric oxide
c) prostacylin synthase
d) thrombomondulin
e) voltage gated K channels

3.A 50 year old woman with COPD comes to physician of 3 month history of progressive shortness of breath. Physical exam shows jugular venous distention. A loud pulmonary component of s2 is heard. Pulmonary function tests shows FEV1:FVC ratio of 20% and a severely decreased diffusing capacity of carbon monoxide. based on these findings which of the following is most likely decreased in patients pulmonary vascular smooth muscle.
a) adventitial collagen matrix deposition
b. cytosolic phosphlipase A2 activity
c endothelial nitric oxide synthase production Correct
d endothelin expression
e) voltage gated calcium channel alpha subunit production

4.6 year old girl is brought to ED by dad cause of 15 min history of shortness of breath. She was diagnosed with throat tumor 3 years ago. Respiration are 32/min. Exam shows nasal flaring. Larygocopy shows multiple raised finger shaped lesions arising from the vocal cords and epiglottis and partially occluding the larynx. The lesions are excised. Micro exam shows finger shaped fibrovascular cores lined with benign squamous epithelium. Which of the following is the most likely virus.
a) CMV
b) EBV
C) HSV1
d)HSV2)
e) HPV type 6 Correct



Answers in bold
 
sec 2 q 42
vancomycin resistant E.feacalis cultured in liquid medium. ten generation are propagated. the culture is inoculated in solid culture medium lacking vancomycin. the resulting colonies are screened for vancomycin resistance. vancomycin-sensitive colonies are observed at frequency of 1 per 100 cell. which of the following mechanisms explains the decreased resistance in these colonies?
a- gene duplication
b- gene insertion
c- plasmid loss
d- point mutation
3- transposition
 
sec 2 q 42
vancomycin resistant E.feacalis cultured in liquid medium. ten generation are propagated. the culture is inoculated in solid culture medium lacking vancomycin. the resulting colonies are screened for vancomycin resistance. vancomycin-sensitive colonies are observed at frequency of 1 per 100 cell. which of the following mechanisms explains the decreased resistance in these colonies?
a- gene duplication
b- gene insertion
c- plasmid loss
d- point mutation
3- transposition
I believe it was plasmid loss-C
 
  • Like
Reactions: 1 user
Patient has septic shock. ADH doesnt work so they give him dopamine. Which receptor stimulation is of most benefit to this patient?
a1, a2, b1, b2, d2
Is it just a1 because vasoconstriction -> increase BP?
 
  • Like
Reactions: 1 users
A previously healthy 48-year-old man comes to the physician because of a 2-month history of increasing abdominal girth and inability to achieve an erection. He has smoked 1 pack of cigarettes daily for 20 years; he also has drunk 1 pint of liquor daily during this period. His temperature is 37°C (98.6°F), pulse is 98/min, respirations are 14/min, and blood pressure is 120/76 mm Hg. Physical examination shows scleral icterus and spider angiomata. The lungs are clear to auscultation. Cardiac examination shows no abnormalities. There is gynecomastia. Abdominal examination shows ascites and a prominent umbilical venous pattern. His testes are small. Which of the following is the most likely cause of this patient's gynecomastia?

D) Failure to the liver to degrade estrogen. -- Is this the correct answer??

Girl has chronic cough with thick sputum, abdominal cramps in RLQ, and frequent resp infections. Clubbing of fingers, hyperresonance on chest percussion. Diffuse crackles and scattered wheezes. On xray - diffuse hyperinflation of the right upper lobe. She has a healthy brother. What's the likelihood he is a carrier for the condition?
1 of 1
1 of 2
1 of 3
1 of 4.... wrong
2 of 3 ... I saw on test pirates that this was the correct answer, can someone explain why? THANKS
 
Last edited:
  • Like
Reactions: 1 users
Girl has chronic cough with thick sputum, abdominal cramps in RLQ, and frequent resp infections. Clubbing of fingers, hyperresonance on chest percussion. Diffuse crackles and scattered wheezes. On xray - diffuse hyperinflation of the right upper lobe. She has a healthy brother. What's the likelihood he is a carrier for the condition?
1 of 1
1 of 2
1 of 3
1 of 4.... wrong
2 of 3 ... I saw on test pirates that this was the correct answer, can someone explain why? THANKS

Cystic fibrosis displays autosomal recessive inheritance. Assuming that both her parents were healthy carriers, they both have genotype "Aa" ("A" signifying normal allele, "a" for mutant allele). Thus, there is a 50% chance dad passes on "A" and a 50% chance he passes on "a"; the same holds true for mom. This allows us to determine that there is a 25% chance the brother would get "aa", a 50% chance he would get "Aa" and a 25% chance he would get "AA". If we stopped here, that would give us 1 out of 4, but that is wrong. The question states that the brother is healthy; this means he did not get "aa", so we can eliminate that. This leaves us 3 genotype possibilities, 2 of which involve him receiving an "a" allele from either mom or dad, while only 1 possibility of him getting "A" from both parents. Thus, 2 out of 3.

Hopefully/surely this question gave you a pedigree chart, or at least some verbal info on the parent's health? Otherwise it's a terrible question. Because, if a mother with cystic fibrosis and a father who was a carrier had a child (or heck, if it was one of the small percentage of CF men who are fertile), then there is a 100% chance (1 of 1) he is a carrier if he doesn't have the disease itself. While you should make the likely assumption both parents are just carriers if they don't tell you otherwise, that's stupid of them if they didn't provide that info.
 
Last edited:
  • Like
Reactions: 1 users
Cystic fibrosis displays autosomal recessive inheritance. Assuming that both her parents were healthy carriers, they both have genotype "Aa" ("A" signifying normal allele, "a" for mutant allele). Thus, there is a 50% chance dad passes on "A" and a 50% chance he passes on "a"; the same holds true for mom. This allows us to determine that there is a 25% chance the brother would get "aa", a 50% chance he would get "Aa" and a 25% chance he would get "AA". If we stopped here, that would give us 1 out of 4, but that is wrong. The question states that the brother is healthy; this means he did not get "aa", so we can eliminate that. This leaves us 3 genotype possibilities, 2 of much involve him receiving an "a" allele from either mom or dad, while only 1 possibility of him getting "A" from both parents. Thus, 2 out of 3.

Hopefully/surely this question gave you a pedigree chart, or at least some verbal info on the parent's health? Otherwise it's a terrible question. Because, if a mother with cystic fibrosis and a father who was a carrier had a child (or heck, if it was one of the small percentage of CF men who are fertile), then there is a 100% chance (1 of 1) he is a carrier if he doesn't have the disease itself. While you should make the likely assumption both parents are just carriers if they don't tell you otherwise, that's stupid of them if they didn't provide that info.

THANK you for the big explanation! there was no additional information...
 
  • Like
Reactions: 1 users
Thanks in advance my exam is next week :S

1. A 52 yo woman with breast cancer comes for follow up. She had 4 week course of radiation treatment 6 months ago. Her respirations are 26/min. PE shows no recurrence of the cancer. CT scan shows bilateral patches of atelectasis in the upper lung fields. The atelectasis in this patient most likely developed because of which of the following primary pathophysiologic processes?
a) compression
b) consolidation
c) Contraction
d) Obstruction
E) Resorption

Math again, someone here knows an easy way to do this
To decrease the risk for cardiovascular disease, a 24 year old man begins a weight loss diet by limiting his daily caloric intake to 2000 calories, in order to maintain the recommended protein intake (56 g daily) a balanced decrease in carbohydrates and fat is required. (caloric ratio of fat to carb 30:55) Which of the following best describes the number of calories that should be provided by fat in this patient's diet each day?
a) 370
b) 430
c) 510
d) 630
e) 740
 
Last edited:
  • Like
Reactions: 1 user
Patient has septic shock. ADH doesnt work so they give him dopamine. Which receptor stimulation is of most benefit to this patient?
a1, a2, b1, b2, d2
Is it just a1 because vasoconstriction -> increase BP?
I thought it was B1 because d1 and b1 share Gs receptor... So confused now, so A1 is the correct answer ?? did you find out why

I found this on medscape ::

Endogenous catecholamine, acting on both dopaminergic and adrenergic neurons

Low dose stimulates mainly dopaminergic receptors, producing renal and mesenteric vasodilation; higher dose stimulates both beta1-adrenergic and dopaminergic receptors, producing cardiac stimulation and renal vasodilation; large dose stimulates alpha-adrenergic receptors
 
Thanks in advance my exam is next week :S

1. A 52 yo woman with breast cancer comes for follow up. She had 4 week course of radiation treatment 6 months ago. Her respirations are 26/min. PE shows no recurrence of the cancer. CT scan shows bilateral patches of atelectasis in the upper lung fields. The atelectasis in this patient most likely developed because of which of the following primary pathophysiologic processes?
a) compression
b) consolidation
c) Contraction (radiation therapy 6 months ago. Now fibrosis is causing it)
d) Obstruction
E) Resorption

Math again, someone here knows an easy way to do this
To decrease the risk for cardiovascular disease, a 24 year old man begins a weight loss diet by limiting his daily caloric intake to 2000 calories, in order to maintain the recommended protein intake (56 g daily) a balanced decrease in carbohydrates and fat is required. (caloric ratio of fat to carb 30:55) Which of the following best describes the number of calories that should be provided by fat in this patient's diet each day?
a) 370
b) 430
c) 510
d) 630 (This has been discussed many times in this thread)
e) 740
56g x 4 cal/g = 224 cal (so he has 2000 - 224 = 1776 left to eat in one day). 630 + 1146 = 1776 and 630/1146 = 30/55

okay! So they told us the person must get atleast 56g of protein in that 2000cal diet. So we multiply 56g into 4 (1gprotein=4cal) and substract it from 2000. We get roughly 1776 calories.
Now they tell us that from that 1776cal, we should maintain the ratio of 30:55 between fat and carb.
But that ratio is BETWEEN fat and carb and not between fat and the TOTAL NO. of calories. To get that ration we will add 30 and 55 which will no give us 85. And 30:85 is our required ratio between fat and the left over calories(1776). Now we just have to multiply that ratio to the actual number of leftover calories(1776) to get the answer.

Hope this is helpful :)
 
2. A 27 YO primigravid woman at 18 weeks gestation comes to physician for routine prenatal examination. The uterus consistent in size with 18 week gestation. Ultrasonography shows a male fetus. The collecting system and pelvis of the left kidney is dilated and the renal cortex appears compressed. The left and right ureters are not dilated. The right kidney appears normal. Amniotic fluid volume is normal. Which causing renal finding in this fetus ?
a. Abnormal ureter insertion into bladder wall
b. Agenesis of upper urinary tract
c. Congenital urethral stricture (WRONG)
d. Ectopic ureter
e. Incomplete recanalization of proximal ureter
f. Posterior urethral valves
Can someone pls tell what's the right answer?
With explanation?:)
 
i may have chosen this answer wrong, please correct me if my intended answer is wrong
A 39 yo male with polycystic kidney disease is brought to the phys for followup. 6 mo history of intermittent blood in his urine. temp 37C pulse 100 resp 24 BP 160/90 and serum creatinine is 8mg/dl. ABG on room air?
PH pCO2 HCO3
A) 7.22 28 11 (is this the answer? )
B) 7.32 64 32
C) 7.38 40 23
D) 7.46 19 13 ( wrong)
E) 7.49 50 37
please explain. thanks
Hey why is there metabolic acidosis? I thought RAAS is what caused the hypertension so you would have alkalosis?
 
Hey why is there metabolic acidosis? I thought RAAS is what caused the hypertension so you would have alkalosis?

PKD can lead to renal failure, because the cysts destroy the parenchyma so RENAL FAILURE = MAD HUNGER
Metabolic Acidosis
Dyslipidemia
Hyperkalemia
Uremia
Na and h2o retention (hypertension)
Growth/develop delay
Erythropoietin failure
Renal osteodistrophy.
 
  • Like
Reactions: 1 user
Can someone confirm the correct answer here
The question about the 52 year old man that played tennis all day and is dehydrated, which of the findings in the nephron best describes the tubular osmolality compared with serum in this patient?
Proximal tubule : Isotonic Macula Densa: hypotonic Medullary Collecting Duct: hypertonic ?
 
Can someone confirm the correct answer here
The question about the 52 year old man that played tennis all day and is dehydrated, which of the findings in the nephron best describes the tubular osmolality compared with serum in this patient?
Proximal tubule : Isotonic Macula Densa: hypotonic Medullary Collecting Duct: hypertonic ?
My thinking:

PCT is isotonic - correct

Macula densa - reads the DCT, which is the most dilute - correct

Medullary collecting duct - ADH absorbs all the water so hypertonic - correct

799b02f64fefbce80f6c3eeacdac979d.png
 
  • Like
Reactions: 3 users
A 20-year-old woman at 27 weeks' gestation is admitted to the hospital because of a 12-hour history of intense uterine contractions occurring every 8 minutes. Her membranes ruptured 32 hours ago. Her temperature is 39.1°C(102.4°F), and pulse is 115/min. Physical examination shows tenderness of the uterus. Pelvic examination shows a closed cervix that is not effaced. The fetal heart rate is 210/min. Which of the following is the primary stimulus for her uterine muscle contractions?

A) Decreased myometrial intracellular calcium

B) Direct response to maternal hyperthermia

C) Inflamed maternal decidua release of prostaglandin

D) Maternal adrenocorticosteroid release

E) Stressed fetal production and release of oxytocin

I know oxytocin and prostaglandins cause contraction... answer isnt E because I picked it and it's wrong... so is the answer C?
 
THANK you for the big explanation! there was no additional information...
A 20-year-old woman at 27 weeks' gestation is admitted to the hospital because of a 12-hour history of intense uterine contractions occurring every 8 minutes. Her membranes ruptured 32 hours ago. Her temperature is 39.1°C(102.4°F), and pulse is 115/min. Physical examination shows tenderness of the uterus. Pelvic examination shows a closed cervix that is not effaced. The fetal heart rate is 210/min. Which of the following is the primary stimulus for her uterine muscle contractions?

A) Decreased myometrial intracellular calcium

B) Direct response to maternal hyperthermia

C) Inflamed maternal decidua release of prostaglandin

D) Maternal adrenocorticosteroid release

E) Stressed fetal production and release of oxytocin

I know oxytocin and prostaglandins cause contraction... answer isnt E because I picked it and it's wrong... so is the answer C?

Yup, C. She possibly got an infection after the membrane rupture, now the inflammation and the consequent pgs are causing contractions.
 
  • Like
Reactions: 1 user
@chillaxbro
Check this out from Uptodate

Preterm premature rupture of membranes
Genital tract infection is the single most common identifiable risk factor for PPROM. Three lines of epidemiologic evidence strongly support this association: (1) women with PPROM are significantly more likely than women with intact membranes to have pathogenic microorganisms in the amniotic fluid, (2) women with PPROM have a significantly higher rate of histologic chorioamnionitis than those who deliver preterm without PPROM, and (3) the frequency of PPROM is significantly higher in women with certain lower genital tract infections (particularly bacterial vaginosis) than in uninfected women.

The association between bacterial colonization of the lower genital tract and PPROM is not surprising. Many of the microorganisms that colonize the lower genital tract have the capacity to produce phospholipases, which can stimulate the production of prostaglandins and thereby lead to the onset of uterine contractions. In addition, the host's immune response to bacterial invasion of the endocervix and/or fetal membranes leads to the production of multiple inflammatory mediators that can cause localized weakening of the fetal membranes and result in PPROM [3]. Genetic regulation of the host's immune and inflammatory response appears to play a role in susceptibility and response to infections associated with PPROM
 
  • Like
Reactions: 9 users
new antiplatelet agent is developed for the prevention of recurrence of stroke. In a large randomized clinical trial with equal numbers of men and women, the rates of stroke are lower in patients receiving the new agent than in patients receiving the standard treatment. Results are shown:
Recurrent Stroke Rates per 1000 Person-Years
Standard Treatment New Antiplatelet Drug
Women .12 .04
Men .24 .08
Overall .18 .06

Based on these results, which of the following is the relative risk reduction in women?

Can someone explain why this is 67?
 
new antiplatelet agent is developed for the prevention of recurrence of stroke. In a large randomized clinical trial with equal numbers of men and women, the rates of stroke are lower in patients receiving the new agent than in patients receiving the standard treatment. Results are shown:
Recurrent Stroke Rates per 1000 Person-Years
Standard Treatment New Antiplatelet Drug
Women .12 .04
Men .24 .08
Overall .18 .06

Based on these results, which of the following is the relative risk reduction in women?

Can someone explain why this is 67?



RRR= 1- relative risk.
Relative risk reduction here will be
Relative risk = new drug/standard = .04/.12 = .33
REDUCTION 1 - .33 = .67



http://medicowesome.blogspot.com/2013/12/bio-statistics-mnemonics-tips-tricks.html
 
  • Like
Reactions: 3 users
Anyone have an explanation for this one?

Fungi expressing B-glucans on their cell surfaces are found to trigger protective innate immune responses. Which of following is most likely to alter this response by decreasing the gluten expression on the fungal cell surface?

A)Amphotericin B
B) Caspofungin
C) Flucytosine
D) Terbinafine
E) Voriconazole
 
Anyone have an explanation for this one?

Fungi expressing B-glucans on their cell surfaces are found to trigger protective innate immune responses. Which of following is most likely to alter this response by decreasing the gluten expression on the fungal cell surface?

A)Amphotericin B
B) Caspofungin
C) Flucytosine
D) Terbinafine
E) Voriconazole
Answer is B. Capsofugin. It is the only one that fits
 
A 21-year old woman comes to the physician because of 10-day history of difficulty walking. Two years ago, she had loss of vision int he left eye which improved over 2 months. Neurologic examination shows decreased visual acuity in the left eye with pallor of optic disc. She has past-pointing on a finger-nose test. She has a broad-based gait. An MRI of the brain shows lesion in the white matter of cerebellum. Which of the following describes the pathogenesis of the disease process in this patient?

A. Antibodies with specificity to axonal microtubules bind to surface of axons
B. CD4+ T lymphocytes are activated by myelin basic protein
C. CD8+ T lymphocytes kill astrocytes
D. Complexes of rheumatoid factor and IgG are deposited in choroid plexus
E. Inflammatory vasculitis affects perforating arterioles
 
A 21-year old woman comes to the physician because of 10-day history of difficulty walking. Two years ago, she had loss of vision int he left eye which improved over 2 months. Neurologic examination shows decreased visual acuity in the left eye with pallor of optic disc. She has past-pointing on a finger-nose test. She has a broad-based gait. An MRI of the brain shows lesion in the white matter of cerebellum. Which of the following describes the pathogenesis of the disease process in this patient?

A. Antibodies with specificity to axonal microtubules bind to surface of axons
B. CD4+ T lymphocytes are activated by myelin basic protein
C. CD8+ T lymphocytes kill astrocytes
D. Complexes of rheumatoid factor and IgG are deposited in choroid plexus
E. Inflammatory vasculitis affects perforating arterioles
My answer was b. Myelin basic protein is associated in the synthesis of myelin in CNS and the question describe a patient with MS, which is an autoimmune demyelinating disease. Hope it helps.
 
  • Like
Reactions: 1 user
a 35 year old women comes to the physician bcz of fever n sharp chest pain for 3 days. temp= 101.3. friction rub heard. al the secondary causes of pericarditis is ruled out. most likely cause of primary pericarditis?
A. bacterium
B. fungus
c. parasite
d. tumor
e. virus
 
Top